Difference between revisions of "2013 Mock AIME I Problems/Problem 14"
m (solution edits, links) |
m (more thorough explanation of why fermat works here) |
||
(One intermediate revision by the same user not shown) | |||
Line 4: | Line 4: | ||
==Solution== | ==Solution== | ||
− | Since <math>997</math> is prime, by [[Fermat's Little Theorem]], we have <math>a_1^{997}+a_2^{997}+\cdots + a_{2013}^{997} \equiv a_1+a_2+\cdots + a_{2013} \pmod{997}</math>, which, by | + | By [[Vieta's Formulas]], the product of the roots is <math>-2014^2</math>. Since <math>997</math> is prime with <math>997\nmid2014^2</math>, all the roots are relatively prime to <math>997</math>. Thus, by [[Fermat's Little Theorem]], we have <math>a_1^{997}+a_2^{997}+\cdots + a_{2013}^{997} \equiv a_1+a_2+\cdots + a_{2013} \pmod{997}</math>, which, by Vieta, equals <math>-4 \equiv 993 \pmod{997}</math>. Thus our answer is <math>\boxed{993}</math>. |
==See also== | ==See also== | ||
+ | *[[2013 Mock AIME I Problems]] | ||
*[[2013 Mock AIME I Problems/Problem 13|Preceded by Problem 13]] | *[[2013 Mock AIME I Problems/Problem 13|Preceded by Problem 13]] | ||
*[[2013 Mock AIME I Problems/Problem 15|Followed by Problem 15]] | *[[2013 Mock AIME I Problems/Problem 15|Followed by Problem 15]] | ||
[[Category:Intermediate Number Theory Problems]] | [[Category:Intermediate Number Theory Problems]] |
Latest revision as of 10:51, 4 August 2024
Problem
Let If are its roots, then compute the remainder when is divided by 997.
Solution
By Vieta's Formulas, the product of the roots is . Since is prime with , all the roots are relatively prime to . Thus, by Fermat's Little Theorem, we have , which, by Vieta, equals . Thus our answer is .